Merge remote branch 'public/master'
[course.git] / latex / problems / Serway_and_Jewett_4 / problem05.52.tex
1 \begin{problem*}{5.52}
2 An amusement park ride consists of a rotating circular platform
3 $d=8.00\U{m}$ in diameter from which $m=10\U{kg}$ seats are suspended
4 at the end of $l=2.50\U{m}$ massless chains (Fig. P5.52).  When the
5 system rotates, the chains make an angle of $\theta=28.0\dg$ with the
6 vertical.
7 \Part{a} What is the speed of each seat?
8 \Part{b} Draw a free-body diagram of a $m_c=40.0\U{kg}$ child riding
9 in a seat, and find the tension in the chain.
10 \end{problem*} % problem 5.52
11
12 \begin{solution}
13 \Part{a}
14 We will eventually use $v=\sqrt{a_c r}$ as we have in all the other
15 problems in this homework assignment to find $v$.
16
17 First, we need to find the radius $r$ of the path that the seat takes
18 around the ride.
19 \begin{equation}
20  r=\frac{d}{2} + l \sin \theta
21   =(4.00 + 2.50\sin 28.0\dg)\U{m}
22   =5.1736\ldots\U{m}
23 \end{equation}
24
25 Now we need to find the centerward acceleration $a_c$.  Drawing a free
26 body diagram of our seat, we see that the only forces acting upon it
27 are the tension \vect{T} and gravity $\vect{F}_g$.  We know that the
28 seat does not rise or fall in the vertical (\vect{y}) direction, so
29 summing the forces we have
30 \begin{align}
31  \sum F_y &= T \cos \theta - mg=m a_y=0 \\
32   T &= \frac{mg}{\cos\theta} \label{eqn.T}\\
33  \sum F_c &= T \sin \theta 
34           =mg\tan\theta
35           =m a_c \\
36  a_c &= g\tan\theta
37     =9.8\U{m/s}^2 \cdot \tan 28.0\dg
38     =5.2108\ldots\U{m/s}^2 \label{eqn.ac}
39 \end{align}
40 So
41 \begin{equation}
42  v=\sqrt{a_c r}
43   =\sqrt{g \tan \theta \cdot (\frac{d}{2} + l \sin \theta)}
44   =\sqrt{ 5.2108\ldots\U{m/s}^2 \cdot 5.1736\ldots\U{m}}
45   =5.19\U{m/s}
46 \end{equation}
47
48 \Part{b}
49 Our free body diagram with a child in the seat will be the same as our
50 diagram from \Part{a} but with a new mass $m'=m + m_c=50\U{kg}$.
51
52 Before we find the tension in the chain, we should check to see if the
53 chain angle changes.  The angular velocity $\omega=v/r$ does not
54 change when people get into the seats (because they are of negligible
55 mass compared to the platform), so we can relate our new velocities
56 $v'$ and $r'$ using the same $\omega$ that we had in \Part{a}.
57 \begin{equation}
58  \omega=\frac{v'}{r'}
59        =\frac{v}{r}
60        =\frac{\sqrt{ g r \tan \theta}}{r}
61         = \sqrt{\frac{g\tan\theta}{r}}
62        =1.00\U{rad/s}
63 \end{equation}
64 Not that the numerical value is important, just that it is a constant.
65 We can plug $v'=\omega r'$ into our centerward acceleration equation
66 \begin{equation}
67  a_c'=\frac{v'^2}{r'}
68     =\frac{\omega^2 r'^2}{r'}
69     =r' \omega^2
70 \end{equation}
71 And applying this to eqn. \ref{eqn.ac}(which hasn't changed except for
72 the need to substitute primed variables)
73 \begin{align}
74   a_c' &= g \tan\theta'=r' \omega^2 \\
75   g \tan\theta' &= \left( \frac{d}{2} + l \sin\theta' \right) \omega^2
76 \end{align}
77 The only unknown in this equation is $\theta'$, but the equation is
78 analytically unsolvable.  We know $\theta'=28.0\dg$ is one solution,
79 because there are no masses in this equation, so is must also hold for
80 case \Part{a}.  Then we have to decide if there will be any other
81 solutions.  We know intuitively that any solutions will have $0\dg <
82 \theta' < 90\dg$.  Considering the $\sin$ and $\tan$ functions on that
83 interval, we see that $\sin\theta'$ is concave down and continuous
84 over the entire interval, and that $\tan\theta;$ is concave up and
85 continuous over the entire interval.  Therefore, the left hand side of
86 this equation only equals the right hand side at a single value of
87 $\theta'$ so our $28.0\dg$ solution is unique.  If this doesn't make
88 sense to you, you can graph the right and left hand sides to check.
89
90 Having proved that $\theta'=\theta$ we can move on to solve for the
91 tension.  Using eqn \ref{eqn.T}.
92 \begin{equation}
93  T'=\frac{m' g}{\cos \theta'}
94    =\frac{50\U{kg} \cdot 9.8\U{m/s}^2}{\cos 28.0\dg}
95    =\ans{555\U{N}}
96 \end{equation}
97
98 As far as grading is concerned I will accept anything where you did
99 any of the following:
100 \begin{itemize}
101 \item assumed $\theta$ didn't change (skipping the whole $\theta'=\theta$ step)
102 \item assumed $\omega$ didn't change, and you went on to show
103   $\theta'=\theta$ is a valid solution (skipping the uniqueness step).
104 \item assumed $\omega$ didn't change, and proved that $\theta'=\theta$
105   is valid and unique.
106 \end{itemize}
107 \end{solution}